1994 AJHSME Problems/Problem 1

Revision as of 14:06, 15 August 2011 by Testingtesting (talk | contribs) (Created page with "==Problem== Which of the following is the largest? <math>\text{(A)}\ \dfrac{1}{3} \qquad \text{(B)}\ \dfrac{1}{4} \qquad \text{(C)}\ \dfrac{3}{8} \qquad \text{(D)}\ \dfrac{5}{12...")
(diff) ← Older revision | Latest revision (diff) | Newer revision → (diff)

Problem

Which of the following is the largest?

$\text{(A)}\ \dfrac{1}{3} \qquad \text{(B)}\ \dfrac{1}{4} \qquad \text{(C)}\ \dfrac{3}{8} \qquad \text{(D)}\ \dfrac{5}{12} \qquad \text{(E)}\ \dfrac{7}{24}$